2006 AMC 12A Problems/Problem 8

Revision as of 21:08, 30 January 2007 by Azjps (talk | contribs) (expand, add succession box)

Problem

How many sets of two or more consecutive positive integers have a sum of $15$?

$\mathrm{(A) \ } 1\qquad \mathrm{(B) \ } 2\qquad \mathrm{(C) \ } 3\qquad \mathrm{(D) \ } 4\qquad \mathrm{(E) \ }  5$

Solution

Notice that if the consecutive positive integers have a sum of 15, then their average (which could be a fraction) must be a factor of 15. If the number of integers in the list is odd, then the average must be either 1, 3, or 5, and 1 is clearly not possible. The other two possibilities both work:

  • 1+2+3+4+5 = 15
  • 4+5+6 = 15

If the number of integers in the list is even, then the average will have a $\frac{1}{2}$. The only possibility is $\frac{15}{2}$, from which we get:

  • 7+8 = 15

Thus, the correct answer is C (3).

See also


{{{header}}}
Preceded by
Problem 7
AMC 12A
2006
Followed by
Problem 9